Expected Value and integration

Click For Summary
To find the expected value E(X) for the number of observations in the interval (0.5, 1) from the pdf f(Y) = 3y^2, the correct approach is to compute the definite integral of the pdf over that interval. The integral E(X) = ∫(0.5 to 1) 3x^2 dx yields a value of 0.75. This value represents the expected proportion of observations in the specified interval, which can then be multiplied by the total number of observations (15) to find the expected count. The confusion arises from the incorrect assumption that the total probability sums to 1 for the entire interval, rather than focusing on the specific range of interest. The final expected count of observations in the interval (0.5, 1) is 11.25.
rhyno89
Messages
18
Reaction score
0

Homework Statement



Suppose that 15 observations are chosen at random for the pdf f(Y) = 3y^2 on the interval 0 to 1. Let X denote the number that lie in the interval (.5,1). Find E(X)

Homework Equations





The Attempt at a Solution



Ok so to get the expected value u integrate the pdf times the random variable.
Doing this is the integral 3y^3 and it turns to .75y^4

This is where I get confused. First of all I thought that it would have summed to 1 so that the number of X on that range would be 15, but it can't be it would be 11.25

Aside from that, to find the number over this range i kno is 15x the expected value but i can't figure out the bounds. Is it the value of 0,1 - the value of .5,1 or something else all together

Thanks in advance
 
Physics news on Phys.org
rhyno89 said:

Homework Statement



Suppose that 15 observations are chosen at random for the pdf f(Y) = 3y^2 on the interval 0 to 1. Let X denote the number that lie in the interval (.5,1). Find E(X)

Homework Equations





The Attempt at a Solution



Ok so to get the expected value u integrate the pdf times the random variable.
Doing this is the integral 3y^3 and it turns to .75y^4

This is where I get confused. First of all I thought that it would have summed to 1 so that the number of X on that range would be 15, but it can't be it would be 11.25

Aside from that, to find the number over this range i kno is 15x the expected value but i can't figure out the bounds. Is it the value of 0,1 - the value of .5,1 or something else all together

Thanks in advance
Try to be more consistent in your use of variables. You are using x and y interchangeably, and that's bound to lead to confusion.

E(x) = \int_{0.5}^1 3x^2 dx

You got the right antiderivative, but you should be working with a definite integral. Since you're concerned only with the interval [0.5, 1], you should expect the expected value to be a number somewhere in that interval.
 
Question: A clock's minute hand has length 4 and its hour hand has length 3. What is the distance between the tips at the moment when it is increasing most rapidly?(Putnam Exam Question) Answer: Making assumption that both the hands moves at constant angular velocities, the answer is ## \sqrt{7} .## But don't you think this assumption is somewhat doubtful and wrong?

Similar threads

Replies
4
Views
2K
  • · Replies 4 ·
Replies
4
Views
3K
Replies
1
Views
1K
  • · Replies 3 ·
Replies
3
Views
1K
  • · Replies 5 ·
Replies
5
Views
2K
  • · Replies 2 ·
Replies
2
Views
2K
  • · Replies 10 ·
Replies
10
Views
3K
  • · Replies 6 ·
Replies
6
Views
2K
Replies
2
Views
2K
  • · Replies 4 ·
Replies
4
Views
2K